NOTEPAD
Results
of 200 questions answered correctly

You have reached of 200 points, ( %)

Your time

Question 1 of 200

1. A 40-year-old woman on examination presents with intensified basal metabolic rate. What hormone present in excess leads to such condition?

Explanation

The Thyroid gland controls the body’s basal metabolic. Histologically, it is made up of follicles, follicular and parafollicular cells. These cells help secrete Triiodothyronine T3, thyroxine T4 and calcitonin.T3 and T4 control basal metabolic rate while calcitonin increases bone calcium level.
2. Autopsy of a woman revealed the following morphologic changes: stenosis of the atrioventricular opening, mitral insufficiency. Histologically there are focal cardiosclerosis and ”blooming” Aschoff nodules in the myocardium. What is the most likely diagnosis?

Explanation

Rheumatism refers to connective tissue disorders eg, arthritis, spondylitis. Biochemically, there is an increase in C-reactive protein; morphologically we see aschoff bodies ( nodules found in the hearts of individuals with rheumatic fever). SLE is characterised by butterfly/malar rash.
3. After a case of sepsis a 27-year-old woman developed ”bronzed” skin discoloration characteristic of Addison’s disease. Hyperpigmentation mechanism in this case is based on increased secretion of:

Explanation

 

‘Bronzed skin’ indicates hyperpigmentation, it is one of the symptoms of addison’s disease. Melanocyte stimulating hormones are released from cells of the pars intermedia in the anterior pituitary gland. They stimulate melanocytes to produce more melanin leading to increased pigmentation.

4. A 16-year-old girl presents with no hair on the pubis and in the armpits, her mammary glands are underdeveloped, no menstruations. What hormone imbalance can it be indicative of?

Explanation

The ovary secretes estrogen and progesterone that helps in the development and maturation of the female sex characteristics ie, development of breast, wide hips etc. In males, testosterone produced from testes carry out this function (male sex characteristics).
5. From the feces of a patient with acute gastroenteritis a pure culture of microorganisms was obtained. The microorganisms are small mobile slightly curved gram-negative bacilli that within 6 hours grow into a light blue film on the 1% alkaline peptone water. Such properties are characteristic of the following microorganism:

Explanation

 
  1. Cholera is caused by Vibrio cholera which is a gram-negative comma shaped bacteria. It is a facultative anaerobe. Its growth medium is 1% alkaline peptone water.

6. A patient was brought into the infectional diseases hospital on the 8th day since the disease onset. The patient complains of headache, malaise, and weakness. A sample of blood was taken for the serological test. Widal agglutination test results with blood sample diluted 1:200 and typhoid fever O diagnosticum were positive. What diagnosis can be made based on the results of this test?

Explanation

Widal agglutination test is a diagnostic method used for the diagnosis of typhoid fever ( enteric fever).
7. Protective function of saliva is based on several mechanisms, including the presence of enzyme that has bactericidal action and causes lysis of complex capsular polysaccharides of staphylococci and streptococci. Name this enzyme:

Explanation

 

The saliva contains lysozyme which has bactericidal effect ( kills bacteria).

8. In the process of hemoglobin catabolism iron is released and then as a part of special transport protein is returned to the bone marrow, to be used again for hemoglobin synthesis. Name this transport protein:

Explanation

Transferrin is a protein responsible for the transport of Iron during hemoglobin synthesis. Haptoglobin are proteins that bind hemoglobin thus, inhibiting its oxidative function. Ceruloplasmin is a protein for copper transport.
9. The first-aid center has received a victim of a traffic accident diagnosed with closed displaced fracture of the middle third of the thigh. For repositioning of bone fragments the patient received 10 ml of 2% dithylinum solution intravenously, which resulted in prolonged period of apnoea and muscle relaxation. What enzyme is defi- cient, resulting in such pharmacogenetic enzymopathy?

Explanation

 

Dithylinum is a muscle relaxant. Its action in the body is prolonged when the enzyme Pseudocholinesterase/ Butyrylcholinesterase is absent. This enzyme breakdown drugs with choline ester bonds.

10. An 18-year-old student presents with enlarged thyroid gland accompanied by accelerated metabolism and increased heart rate. These signs can be observed during hypersecretion of thyroxin. What organelles of thyroid cells are primarily responsible for hormone production and secretion?

Explanation

 

The Thyroid gland controls the body’s basal metabolic. Histologically, it is made up of follicles, follicular and parafollicular cells. These cells help secrete Triiodothyronine T3, thyroxine T4 and calcitonin.T3 and T4 control basal metabolic rate while calcitonin increases bone calcium level. The Golgi apparatus are vesicles present in the cytoplasm, they take part in secretion and intracellular transport. Ribosomes are for synthesis of protein, mitochondria produce ATP.

11. A patient complains of pain in the upper umbilical region. On palpation there is a mobile painful intestine. What intestine is being palpated by the doctor?

Explanation

The transverse colon is found in the  umbilical region. The sigmoid colon is found at the left iliac region.  
12. Microscopy of the puncture sample obtained from the inflammation focus of the patient with cutaneous abscess revealed numerous blood cells of different types. What cells are the first to transfer from vessels to tissues during inflammation?

Explanation

Neutrophils are the first cells to react to an area of inflammatory area due to their high amount and increased proliferation rate. Eosinophils are increased in allergy.
13. A laboratory experiment on a dog was used to study central parts of auditory system. One of the mesencephalon structures was destroyed. The dog has lost the orienting response to auditory signals. What structure was destroyed?

Explanation

The quadrigeminal plate is a part of the brain responsible for orientational reflexes ( light and sound signals). It consists of: - the Inferior colliculi (posterior quadrigeminal bodies) -for sound - superior colliculi (anterior quadrigeminal bodies)- for light signals. The substantia nigra produces dopamine and the red nucleus controls tone of flexors and extensors.
14. Histological investigation of the uterine scrape of the 45-year-old woman with disturbed ovarian menstrual cycle revealed increased number of endometrial glands, some of which are serrated, while others are dilated and cyst-like. Make the diagnosis:

Explanation

 

Endometrial cystic glandular hyperplasia is characterised by large cystically dilated glands of different sizes found on the wall of the endometrium.

In atypical endometrial hyperplasia,there is presence of atypical cells in the hyperplastic endometrium. Polyps are fingerlike structures that appear as a result of chronic irritation.

15. A 45-year-old man diagnosed with hepatic cirrhosis and ascites underwent drainage of 5 liters of fluid from his abdominal cavity, which resulted in development of syncopal state due to insufficient blood supply to the brain. What circulatory disorder occurred in the abdominal cavity in this case?

Explanation

Arterial hyperemia refers to an increased supply of oxygenated blood to a particular area of the body. Decreased drainage of venous blood will lead to venous hyperemia. As a result of increased blood flow to the abdominal organs, the brain was deficient of arterial blood supply.
16. During the first year of life an infant presents with disturbed process of breast milk curdling. What cells of the proper gastric glands are functionally disturbed?

Explanation

Rennin (chymosin) is a proteolytic enzyme responsible for the curdling of milk. It is produced by the chief cells ( main exocrine) glands in the stomach. The parietal cells produce Hydrochloric acid. This is different from Renin ( produced in the kidney and participates in renin-angiotensin-aldosterone mechanism)
17. A lab rat has subcutaneously received mercury(II) chloride in the amount of 5 mg/kg. 24 hours later the plasma creatinine concentration increased several times. What mechanism of retention azotemia is observed in this case?

Explanation

A rise in plasma creatinine levels indicates a reduction in kidney function. Due to the inability of the kidney to filter out creatinine, the creatinine level will be on the rise. A creatinine clearance test is one of the diagnostic tests run to check the proper functioning of the kidneys.
18. Collagenosis patients typically present with connective tissue destruction processes. The presence of these processes can be confirmed by the increase in:

Explanation

Collagen is a structural protein that makes up connective tissues in the body. Collagenosis refers to the breakdown of collagen fibres. In this, there is an increased amount of oxylysine and oxyproline (amino acids that make up collagen fibres) in the blood. Creatinine is seen in increased breakdown of muscle fibres
19. A 63-year-old man suffers from esophageal carcinoma, presents with metastases into the mediastinal lymph nodes and cancerous cachexia. What pathogenetic stage of neoplastic process is observed in the patient?

Explanation

Initiation- is the first step in cancer development. Initiators, if not already reactive with DNA, are altered via drug-metabolizing enzymes in the body and are then able to cause changes in DNA (mutations).

Promotion- Once a cell has been mutated by an initiator, it is susceptible to the effects of promoters. These compounds promote the proliferation of the cell, giving rise to a large number of daughter cells containing the mutation created by the initiator.

Progression- refers to the stepwise transformation of a benign tumor to a neoplasm and to malignancy.  An increased growth rate, invasiveness, metastasis and an alteration in biochemistry and morphology are present in this phase.

 
20. After an X-ray examination of the tuberculosis clinic patient, he was diagnosed with tumor of the right lung. During operation the surgeon removed the middle lobe of the patient’s right lung. This lobe includes:

Explanation

The right lung is made up of 3 lobes ( superior, middle and inferior); the left is has 2 (superior and inferior). The middle lobe of the right lung has only 2 segments (medial and lateral).

21. A 2-year-old child presents with physical retardation and frequent pneumonias. The child was diagnosed with non-closure of the arterial canal. Hemodynamics disturbance in this case is caused by communication between the:

Explanation

The ductus arteriosus or the Botallo’s duct is a vessel in fetal circulation that connects the pulmonary artery to the aorta ( descending aorta). It closes right before birth and becomes the ligamentum arteriosum.
22. A child diagnosed with purulent inflammation of the middle ear was brought to the otolaryngology department. The disease started with the inflammation of the nasopharynx. It was determined that the infection had reached the tympanic cavity through the eustachian tube that is located in the:

Explanation

[caption id="attachment_1117" align="alignnone" width="300"] Canalis musculotubarius is a canal that runs from the petrous part of the temporal bone down to the tympanic cavity. It is divided into the auditory tube and the canal for the tensor tympani. The eustachian or pharyngotympanic tube (which connects the middle ear to the nasopharynx) lies here.[/caption]
23. During the prestart period an athlete develops increased frequency and force of cardiac contractions. These changes are caused by intensification of the following reflex responses:

Explanation

 

Unconditioned reflexes refers to inborn reflexes ie, reflexes not learned eg. accommodation reflex. Conditioned reflexes are learned over time ( through training).

Sympathetic nervous system is a state of fight/flight/action. Parasympathetic is a state of rest.

A trained athlete in a race indicates conditioned sympathetic reflex.

24. Due to trauma the patient has lost 25% of circulating blood volume. Name the emergency compensatory mechanism against blood loss:

Explanation

Due to the loss of blood volume, interstitial fluid flows into the vessels to increase blood volume.
25. ECG analysis of the patient shows that the T waves are positive in the second standard limb lead and their amplitude and duration is normal. The conclusion can be made that the following process occurs normally in the patient’s ventricles:

Explanation

[caption id="attachment_1118" align="alignnone" width="300"] P-wave indicates Atrial depolarization, QRS complex- ventricular depolarization, T-wave- ventricular repolarization.[/caption]
26. A patient demonstrates sharp decrease of pulmonary surfactant activity. This condition can result in:

Explanation

 

Surfactant is a substance produced by alveolar type II cells. They prevent alveolar tendency to recede (collapse). Alveolar macrophages are responsible for engulfing forein bodies in lungs eg, dust.

27. After a case of common cold the patient developed numbness of the right side of the face. Examination revealed disturbed pain and thermal sensitivity in the right half of the face. What nerve was damaged?

Explanation

 

The Trigeminal nerve, cranial nerve V is a mixed nerve. It gives off three branches:

    1. Opthalmic nerve - sensory nerve, leaves the skull through the superior orbital fissure.

    2. Maxillary nerve- sensory nerve, leaves the skull through through the foramen rotundum

    3. Mandibular nerve- mixed nerve, passes through the foramen ovale. Innervates the frontal ⅔ of the tongue for thermal sensation.

 
28. X-ray exmination of a 57-year-old man indicates local areas of hard bone tissue resorption in some of the patient’s bones. These changes can be associated with increased activity of:

Explanation

Osteoclasts are cells that are responsible for bone resorption (osteoclast activities). Acid phosphatase is a marker for this activity. Osteoblasts are cells for deposition of bone tissue ( osteoblast activities). Alkaline phosphatase is a marker for this activity.
29. A patient presents with acute attack of cholelithiasis. Laboratory examination of the patient’s feces will show the following in this case:

Explanation

Stercobillin is simply a bile pigment gotten at the end of heme breakdown. It is responsible for the brown color of faeces. A Negative reaction indicates absence of stercobilin ie, a pale faeces.

 
  • Jaundice is a disease characterised by the yellow discolouration of skin and mucous membrane due to a build up in bilirubin levels in the blood. Three types exist;

  • Prehepatic/ hemolytic - occurs due to increased breakdown of RBC eg, hemolytic disease of newborn. There is an increased level of unconjugated bilirubin. Stool and urine colour are normal.

  • Hepatic/ parenchymatous  - Occurs in diseases affecting the liver parenchyma eg, cirrhosis, hepatitis etc. There is an increase in both unconjugated and conjugated bilirubin. Urine appears dark and faeces pale.

  • Post hepatic/ obstructive - pathology lies after conjugation of bilirubin and is caused by obstruction of biliary path. Urine is dark and faeces pale. Seen in disease such as cholelithiasis.

30. During autopsy of a 34-year-old man, who died of chronic kidney failure due to renal amyloidosis, in the lungs (mainly in the lower lobes) the pathologist detected multiple bronchial dilations filled with purulent masses accumulated in the bronchial lumen. Surface of the lungs section has fine-meshed pattern and resembles a honeycomb. Histologically there is a chronic inflammation detected in the bronchial wall, muscle fibers are replaced with connective tissue. These changes in the lungs can be defined as:

Explanation

Bronchiectasis is a chronic necrotizing infection of the bronchi and bronchioles characterized by an abnormal dilation of the airways. Histologically we see a chronic inflammation of the walls of the bronchi and bronchioles associated with desquamation of lining epithelium.

Chronic bronchitis presents with a lot of goblet cells filled with mucus.

In lung abscess, we see cavities containing exudates.

 
31. Initial inoculation of water in 1% peptone water resulted in growth of a thin film on the medium surface in 6 hours. Such cultural properties are characteristic of causative agent of the following disease:

Explanation

Cholera is a disease caused by vibrio cholera. A gram negative comma shaped bacillus cultured in alkaline peptone water. They do not form spores nor have capsules.
32. A 30-year-old man complains of suffocation, heaviness in the chest on the right, general weakness. Body temperature is 38.9oC. Objectively the right side of the chest lags behind the left side during respiration. Pleurocentesis yielded exudate. What is the leading factor of exudation in the patient?

Explanation

 

Pleurocentesis is a procedure used to remove air or fluid from the pleural space. Collection of exudate from this process indicates an increase in surrounding vessels permeability.

33. A 42-year-old man with gout presents with high content of uric acid in blood. The patient was prescribed allopurinol to lower the concentration of uric acid. Allopurinol is a competitive inhibitor of the following enzyme:

Explanation

 

Gout is a disease caused by increased production and accumulation of uric acid. Allopurinol is a drug indicated for its treatment. It competitively inhibits the receptors of Xanthine oxidase( an enzyme responsible for the conversion of hypoxanthine and xanthine into Uric acid).

34. Autopsy revealed a large wedge-shaped patch of a dense dark red tissue with clear margins in the upper lobe of the right lung. Histological examination detected there necrosis of the alveolar walls; the alveolar lumen is tightly packed with erythrocytes. What process occurred in the lungs?

Explanation

An infarction is an area of ischemic necrosis within a tissue or an organ. Types include White (ischemic), Red ( hemorrhagic) and white with hemorrhagic halo. The hemorrhagic form is characterised by packed red blood cells.
35. A patient complains of dizziness, thirst, difficult swallowing, and impaired vision of close objects. Objectively: respiratory rate is increased, pupils are dilated, general agitation, talkativeness, though the speech is indistinct. BP is 110/70 mm Hg, heart rate is 110/min. Given symptoms can indicate overdosage of the following drug:

Explanation

 

Atropine is a muscarinic blocker, It causes mydriasis (pupil dilation),  increased heart and breathing rate. Morphine is an opioid analgesic indicated in pain syndrome etc. Aminazine is an antipsychotic drug.

36. A patient presents with indigestion, stomachaches, and excessive salivation. Similar simptoms had already been observed in this patient previously. Laboratory analysis detected oval eggs covered with lumpy capsules in the patient’s feces. What is the most likely cause of the patient’s disorder?

Explanation

Oval eggs covered with capsules indicate ascaris lumbricoides ( causative agent of ascariasis). Fascioliasis is caused by Fasciola hepatica and is gotten from drinking infected water eg, from pond. Enterobiasis is characterised by white/colourless filiform eggs in a child’s underwear. Diphyllobothriasis is gotten from eating raw or undercooked fish.
37. Regional lymph nodes surrounding an infected wound are enlarged. Histological examination shows increased number of macrophages, lymphocytes, and lymphatic follicles, as well as a large amount of plasma cells, in the cortical layer of the lymph nodes. What process in the lymph nodes is indicated by these histologic changes?

Explanation

Antigen refers to foreign bodies that stimulates an immune response. When an antigen gets into the human body, it triggers a response from the body’s immune system, this includes; an increased proliferation and infiltration of cells eg, neutrophils,lymphocytes and macrophages.
38. During removal of the hyperplastic thyroid gland of a 47-year-old woman, the parathyroid gland was damaged. One month after the surgery the patient developed signs of hypoparathyroidism: frequent convulsions, hyperreflexia, laryngospasm. What is the most likely cause of the patient’s condition?

Explanation

 

The parathyroid glands are four structures found on the edges of the thyroid gland. They can be damaged in thyroid surgeries. Histologically, they contain Chief and oxyntic cells; The chief cells are responsible for the secretion of Parathormone which increases blood level calcium. In its absence, calcium level in the blood becomes low leading to symptoms such as convulsions, spasms etc.

39. On examination the patient presents with hirsutism, moon-shaped face, stretch marks on the abdomen. BP is 190/100 mm Hg, blood glucose is 17.6 mmol/L. What pathology is such clinical presentation characteristic of?

Explanation

Adrenocortical hyperfunction refers to Cushing’s disease. It presents with the following symptoms; moon face, hirsutism ( male pattern hair growth), buffalo hump, abdominal striae etc. N/B The adrenal cortex is made of 3 zones namely;

Zona Glomerulosa: Mineralocorticoids eg. aldosterone

Zona Fasciculata; Glucocorticoids

Zona Reticularis: Androgens

 
40. A 45-year-old woman presents with breast cancer. Metastases can spread in this case to the following regional lymph nodes:

Explanation

 

A breast cancer is likely to spread through the axillary nodes.

41. A 40-year-old pregnant woman underwent amniocentesis. Examination determined the fetal karyotype to be 47, XY+21. What fetal pathology was detected?

Explanation

Trisomy 13- patau’s syndrome, trisomy 21- Down’s syndrome, Monosomy X- Turner’s syndrome, 47XXY- kleinfelter’s syndrome, deletion of chromosome 22q11.2- DiGeorge syndrome.
42. A patient used an indirect-acting adrenergic agonist to treat rhinitis. After the patient has been putting in the nose drops for several days, the vasoconstrictive effect of the drug gradually diminished. Name this phenomenon:

Explanation

Tachyphylaxis refers to the gradual reduction in effectiveness of a drug on continuous intake.

Idiosyncrasy refers to an adverse/changed effect of a drug in a person ie, different from the normal effect of this drug. It is most times observed in absence of certain enzymes.

Cumulation (material form) is observed in continuous intake of drugs such as digoxin.

43. In a township there was registered an outbreak of hepatitis, which had supposedly spread through the water supply. What hepatitis virus could be the cause of the outbreak in this township?

Explanation

 

Hepatitis A and E are transmitted through fecal-oral route, Hepatitis B and C are transmitted through blood and other body fluids.  Hepatitis D depends on hepatitis B.

44. A medical student was hospitalized into the infectious diseases unit on the 2nd day after the disease onset; the patient is suspected to have infectious mononucleosis. What results of laboratory analysis can confirm this diagnosis immediately on the day of the hospitalization?

Explanation

Infectious mononucleosis is a disease caused by the Epstein Barr Virus. IgM are antibodies that indicate a recent infection. IgM  to the above virus helps confirm the diagnosis of infectious mononucleosis.
45. A 64-year-old woman presents with disturbed fine motor function of her fingers, marked muscle rigidity, and tremor. The neurologist diagnosed her with Parkinson’s disease. What brain structures are damaged resulting in this disease?

Explanation

 

Parkinson’s disease is a disorder of the nervous system  caused by decreased synthesis in dopamine ( a neurotransmitter). Dopamine is produced from the substantia nigra ( black substance) found in the brain. Symptoms include; tremors, stiffness in movements etc. Levodopa is used in treatment.

46. A 20-year-old young man with tall stature, asthenic body type, signs of hypogonadism and gynecomastia, and low sperm count (azoospermia) has karyotype 47, XXY. What hereditary syndrome can be characterized by this chromosomal anomaly?

Explanation

47XXY- kleinfelter’s syndrome, trisomy 21- Down’s syndrome,  Monosomy X- Turner’s syndrome, Wiskott aldrich syndrome is an X-linked recessive disorder characterised by eczema, thrombocytopenia and low level of IgM immunoglobulin.
47. Pathogenic staphylococcus was obtained from the purulent wound of the patient. Its antibiotic sensitivity was determined to be as follows: penicillin growth inhibition zone - 8 mm; oxacillin - 9 mm, ampicillin - 10 mm, gentamicin - 22 mm, lincomycin - 11 mm. What antibiotic should be chosen for treatment in this case?

Explanation

Gentamicin should be the chosen drug because it created the highest inhibition zone.
48. A patient presents with dilated blood vessels of the anterior medial surface of the lower leg. This condition resulted from the dilation of the following blood vessel:

Explanation

V. saphena magna runs along the anterior medial surface of the thigh, the V. saphena parva runs along the posterior surface of the thigh.
49. A patient, who had received a thermal burn, developed painful boils filled with turbid liquid on the skin. What morphological type of inflammation has developed in the patient?

Explanation

Serous inflammation- marked by a thin/ turbid fluid production eg, boil,blisters

Diphtheric form contains grey-like substance, fibrin which bleeds when removed.

Catarrhal/ suppurative form contains thick mucus production.

 
50. Nitrogen is being excreted from the body mainly as urea. When activity of a certain enzyme in the liver is low, it results in inhibition of urea synthesis and nitrogen accumulation in blood and tissues. Name this enzyme:

Explanation

In urea/ ornithine cycle, Ammonia (NH3) is converted to carbamoyl phosphate by the enzyme carbamoyl phosphate synthase. N/B Ammonia is formed by a combination of nitrogen with 3 molecules of hydrogen.
51. Coronary artery thrombosis resulted in development of myocardial infarction. What mechanisms of cell damage are leading in this disease?

Explanation

52. People, who for a long time remained in hypodynamic state, develop intense pain in the muscles after a physical exertion. What is the most likely cause of this pain?

Explanation

 

Anaerobic glycolysis (breakdown of glucose in the absence of oxygen) mostly takes place during physical activities eg, exercise. During this process, 2 ATP (per glucose molecule) and Lactic acid are formed. An accumulation of lactic acid in the muscles causes intense pain.

53. Histological specimen shows organ parenchyma to consist of lymphoid tissue that forms lymph nodules; the nodules are located diffusely and have a central artery. What anatomical structure has such morphological characteristics?

Explanation

Histologically, spleen has a centrally located artery.
54. A 25-year-old woman complains of visual impairment. Examination revealed disturbed eye accommodation, the pupil is dilated and unresponsive to light. What muscles are functionally disturbed in this case?

Explanation

 

The Iris/pupil sphincter muscle is responsible for the construction of the pupil, the dilator muscle is responsible for dilation. The ciliary muscle controls accommodation while the 4 rectus muscles ( superior, inferior, lateral and medial) control eyeball movements.

55. A patient with hypochromic anemia has hair with split ends and suffers from hair loss. The nails are brittle. Gustatory sensations are affected. What is the mechanism of development of these symptoms?

Explanation

Anemia implies low amount of red blood cells. Iron deficiency anemia also known as hypochromic anemia  is characterised by low color index of RBC and is associated with symptoms relating to iron deficiency eg, brittle nails. B12 deficiency anemia is usually associated with stomach related issues eg, Gastritis, gastrectomy etc; this is because the stomach contains the intrinsic factor of castle which is necessary for B12 absorption
56. A patient suffers from hepatic cirrhosis. What substance excreted in urine should be analyzed to characterize the antitoxic function of liver?

Explanation

Hippuric acid is a normal constituent of urine. Although it is increased on exposure to substances such as toluidine. It is sometimes used as a biomarker to check the antitoxic function of the liver.
57. A 65-year-old woman, who had been suffering from deep vein thrombophlebitis of the lower leg, suddenly died when awaiting her appointment with the doctor. Autopsy revealed loose friable red masses with corrugated dull surface in the main pulmonary artery and its bifurcation. What pathologic process was discovered by the pathologist in the pulmonary artery?

Explanation

Fat/adipose embolism is seen in fractures. It occurs when fat tissues enters the bloodstream. Gas or air embolism is caused by air bubbles or gas in the blood, it occurs in decompression. Thromboembolism is seen when a thrombus/clot leaves its place of formation and is deposited in another part of the body causing an obstruction.
58. Blood of the patients with diabetes mellitus shows increased content of free fatty acids. Name the most likely cause of this:

Explanation

 

Diabetes mellitus is a disease characterised by high levels of glucose in the blood . Tissue uptake of glucose is impaired this leads the body to search for an alternative source of energy, increasing the breakdown of other metabolites such as Fats. Triglyceride lipase is an enzyme that catalyzes the hydrolysis of fats.

59. Representatives of a certain human population can be characterized by elongated body, height variability, decreased volume of muscle mass, increased length of limbs, decreased size and volume of rib cage, increased perspiration, decreased indices of base metabolism and fat synthesis. What type of adaptive evolution is it?

Explanation

Adaptation refers to changes that occur in species over a period of time that helps them fit into their environment. An elongated body, height variability, decreased volume of muscle mass, increased length of limbs, decreased size and volume of rib cage, increased perspiration, decreased indices of base metabolism and fat synthesis indicate a tropical form of adaptation.
60. A 56-year-old man complains of thirst and frequent urination. The endocrinologist diagnosed this patient with diabetes mellitus and prescribed him glibenclamide. What mechanism of action does this drug have?

Explanation

The Pancreas is an organ that has both exocrine and endocrine function. The endocrine part is carried out by a group of cells located in an area called the Islets of Langerhans. These cells include:

  1. Alpha cells- produce glucagon

  2. Beta cells- produce Insulin

  3. Delta cells- produce somatostatin

 

Glibenclamide is a drug indicated in Type 2 diabetes mellitus. It stimulates Beta cells to produce Insulin.

61. Patients with bile duct obstruction typically present with inhibited blood clotting and develop hemorrhages due to insufficient assimilation of vitamin:

Explanation

Bile is responsible for the emulsification of fats which includes fat soluble vitamins (A,D,E & K). Vitamin K helps in blood clotting.
62. A test animal receives electrical impulses that irritate the sympathetic nerve that innervates blood vessels of the skin. What reaction will it cause in the blood vessels?

Explanation

 

Sympathetic nervous system is a branch of the autonomic nervous system. When stimulated it causes; vasoconstriction, pupil dilation, increased heart and breathing rate etc.

63. Examination of the coronary arteries revealed atherosclerotic plaques with calcinosis that narrow the arterial opening by 1/3. In the muscle there are numerous whitish layers of connective tissue. Name the process detected in the myocardium:

Explanation

Closure of  1⁄3rd of arterial opening by atherosclerotic plaques indicates diffuse cardiosclerosis. A closure of 2⁄3rd  or more by athrosclerotic plaques and presence of an area of infarct refers to the morphological description for myocardial infarction.

 
64. A person has increased pulmonary ventilation due to physical exertion. What indicator of external respiration will be significantly increased compared to the resting state?

Explanation

 

In this case, respiratory volume refers to the Tidal volume which is the amount of air inhaled or exhaled at one cycle. Vital capacity of the lungs is the maximum amount of air exhaled after maximum inhalation.

Inspiratory reserve volume is the maximum amount of air that can be forcibly inhaled after normal inspiration (tidal volume). Expiratory reserve volume is the maximum amount of air that can be forcibly exhaled after normal expiration (tidal volume).

65. The carotid bodies on both sides were removed in a test animal. Which of the listed factors WILL NOT be able to cause hyperventilation in the test animal?

Explanation

The carotid bodies are chemoreceptors found at the bifurcation of the carotid artery. It detects changes in the blood flowing through the carotid artery. Amongst these changes, Low partial pressure of oxygen is the main change noticed. Others include, high partial pressure of CO2, and a change in pH and temperature.
66. Due to prolonged stay in the mountains at the altitude of 3000 m above the sea level, a person developed increased oxygen capacity of blood, which was directly caused by intensified production of:

Explanation

High altitude dwellers have an increased need for oxygen. They experience an increase in RBC amount which is stimulated by Erythropoietin (produced in the Kidneys).
67. A woman has been limiting the amount of products in her diet to lose some weight. 3 months later she developed edemas and her diuresis increased. What dietary component deficiency is the cause of this?

Explanation

Edemas are caused by i) Increase in hydrostatic pressure 

                              ii) decrease in oncotic  pressure ( maintained mainly by Albumins)

                              iii) change in osmotic pressure

 

Albumins are the major proteins that maintain oncotic pressure.

68. A microslide of the lung tissue sample taken from a patient with pneumonia shows damage to the cells that carry out respiratory function. What cells of the alveolar wall are damaged?

Explanation

Surfactant are produced by type 2 alveolocytes. They prevent alveolar’s tendency to recede (collapse). Alveolar macrophages are responsible for engulfing inhaled foreign bodies eg, dust.
69. After a psychic trauma a woman developed periodical increases in her blood pressure accompanied by headache, palpitations, and general weakness. What mechanism of hypertension development does this woman have?

Explanation

 

Hypertension is a multifactorial disease that is characterised by increased blood pressure. Essential hypertension is caused by increased peripheral resistance.The increased peripheral resistance in established hypertension is mainly attributable to structural narrowing of small arteries and arterioles, although a reduction in the number or density of capillaries may also contribute.

70. An electron micrograph of a nephron segment shows cuboidal cells with ciliated lining on their apical surfaces; their basal surfaces have basal striation with mitochondria located between the cytolemma invaginations. Name the described nephron segment:

Explanation

Histologically,The Proximal part of nephron has cuboidal epithelium with brush borers(ciliated lining)

 

Distal part has just cuboidal epithelium with no brush border.

71. A patient has been taking bisacodyl for a long time to treat chronic constipation. However, several weeks later the aperient effect of the drug diminished. What is the possible cause of this?

Explanation

Tolerance refers to a reduced effect of a drug on a person due to repeated intake. Cumulation refers to drug accumulation as a result of continuous intake.
72. A force generated by the muscle is not enough to lift a load. What type of muscle contraction occurs in this case?

Explanation

 

Isometric contraction is a type of muscle contraction in which muscle length doesn’t change. In this case, the individual is attempting to lift a load beyond his strength which implies no change of muscle length despite any effort applied.Isotonic contractions maintain constant tension in the muscle as the muscle changes length. This can occur only when a muscle’s maximal force of contraction exceeds the total load on the muscle.

73. Condition of a patient with diabetes mellitus sharply deteriorated after a regular injection of insulin. The patient became anxious and broke out in cold sweat; tremor of the extremities, general weakness, and dizziness appeared. What medicine can remove these symptoms?

Explanation

As a result of insulin administration, the person became hypoglycemic which resulted in the presented symptoms. Of all the drugs listed, Adrenalin is the best possible choice, it stimulates gluconeogenesis and increases blood sugar level.
74. Microscopy of the myocardium of a girl, who died of diphtheria due to heart failure, shows fatty degeneration, multiple foci of cardiomyocytes necrosis, and small foci of cellular infiltrates in the interstitium. What type of myocarditis is it?

Explanation

75. A schizophrenia patient was prescribed aminazine. What pharmacodynamic action of this drug is the grounds for its prescription in this case?

Explanation

Aminazine is an antipsychotic drug primarily used to manage psychosis, principally in schizophrenia and bipolar disorder.
76. A 40-year-old man developed skin redness and swelling in the neck area, where eventually a small abscess appeared. On section the focus is dense and yellowgreen colored. In the purulent masses there are white granules. Histologically there are fungal druses, plasma and xanthome cells, and macrophages detected. Specify the most correct etiological name of this pathological process:

Explanation

A furuncle is an acute inflammation that grows around a hair follicle and contains pus; simply known as a boil. A carbuncle develops under the skin. Morphologicaly, actinomycosis presents with fungal druses, plasma and xanthome cells.
77. A patient was hospitalized into the infectious diseases unit on the 11th day since the disease onset and provisionally diagnosed with typhoid fever. What biological material should be collected from the patient for the analyzes at this stage?

Explanation

For diagnosis of typhoid fever, blood serum should be used for analysis.
78. A man with ischemic heart disease has been taking his medicine too often throughout a day, which resulted in poisoning. Examination detects cyanosis of skin and mucosa, sharp drop of blood pressure, tachycardia, and respiratory depression. Blood methemoglobin is high. What type of medicine did the patient overdose on?

Explanation

 

Organic nitrates eg, Nitroglycerin are drugs which cause vasodilation. They are indicated in diseases such as Angina pectoris. An overdose of this group of drugs can result in cyanosis, B.p drop, tachycardia etc.

79. General structure of eukaryotic genes is as follows: exon-intron-exon. Such functional structure of a gene leads to certain specifics of the transcription process. What sequence will correspond with precursor mRNA (immature)?

Explanation

 

During transcription, the entire gene is copied into a pre-mRNA, which includes exons and introns.An exon is the portion of a gene that codes for amino acids.The parts of the gene sequence that are expressed in the protein are called exons, because they are expressed, while the parts of the gene sequence that are not expressed in the protein are called introns. The sequence Exon-intron-exon corresponds to a precursor mRNA.

80. Atria of a test animal were superdistended with blood, which resulted in decreased reabsorption of Na+ and water in renal tubules. This can be explained by the effect of the following factor on the kidneys:

Explanation

Natriuretic factor or atrial natriuretic peptide is a peptide hormone produced in the atrium of the heart. It decreases sodium reabsorption and increases glomerular filtration, it is a vasodilator and inhibits renin secretion from kidneys. Aldosterone is an adrenal cortex hormone it stimulates sodium reabsorption in the renal tubules. Vasopressin is a posterior pituitary hormone that increases water reabsorption in the renal tubules.
81. A patient with clinical signs of a primary immunodeficiency has functionally disturbed mechanism of antigen-presentation to the immunocompetent cells. What cells are likely to have structural defects?

Explanation

Antigen presenting cells are immune cells that enable a T lymphocyte (T cell) to recognize an antigen and mount an immune response against the antigen. They include Monocytes, macrophages, dendritic cells, etc.
82. During intensive muscle work there is a large amount of ammonia produced in the muscles. What amino acid plays the main role in the transportation of ammonia to the liver and participates in gluconeogenesis reactions?

Explanation

Alanine is an amino acid, it is important for the transport of Ammonia from the muscles to the liver, it is also a precursor for gluconeogenesis.
83. Encephalopathy has developed in a child with hemolytic disease of the newborn. What substance had increased in the child’s blood, resulting in damage to the CNS?

Explanation

Jaundice is a disease characterised by the yellow discolouration of skin and mucous membrane due to a build up in bilirubin levels in the blood. Three types exist; 

 
  • Prehepatic/ hemolytic - occurs due to increased breakdown of RBC eg, hemolytic disease of the newborn. There is an increased level of unconjugated bilirubin. Stool and urine colour are normal.

  • Hepatic/ parenchymatous  - Occurs in diseases affecting the liver parenchyma eg, cirrhosis, hepatitis etc. There is an increase in both unconjugated and conjugated bilirubin. Urine appears dark and faeces pale.

  • Post hepatic/ obstructive - pathology lies after conjugation of bilirubin and is caused by obstruction of biliary path. Urine is dark and faeces pale. Seen in disease such as cholelithiasis.

84. Renal examination shows the kidney to be swollen and plethoric, with easily removable capsule. Renal pelvis and calyces are dilated and filled with turbid urine; their mucosa is dull and has hemorrhagic foci. On section the renal tissue is variegated, there are yellow-gray areas surrounded with plethoric and hemorrhagic zone. What disease can be characterized by such results of macroscopic examination?

Explanation

 

Pyelonephritis is a renal disorder  affecting tubules, interstitium and renal pelvis. In the acute form, the kidneys show variable small yellowish areas surrounded with a hemorrhagic zone. Nephrolithiasis refers to kidney stones. Acute glomerulonephritis is characterised by inflammatory changes in the glomeruli.

85. A 54-year-old man complains of general weakness, frequent colds, and bruises constantly appearing on his body. Blood test: erythrocytes - 2.5 · 1012/L; Hb- 80 g/L; color index - 0.9; reticulocytes - absent; platelets - 50 · 109/L; leukocytes - 58 · 109/L; leukogram: basocytes - 5%, eosinophils - 15%, myeloblasts - 6%, myelocytes - 10%, juvenile - 18%, stab neutrophils - 26%, segmented neutrophils - 10%, lymphocytes - 8%, monocytes - 2%, ESR - 40 mm/hour. What hematologic conclusion can be made?

Explanation

Chronic myelogenous leukemia is a form of leukemia caused by translocation in the philadelphia chromosome (chromosome 22). It is characterised by proliferation in myeloid cells. In acute myeloid leukemia, Myeloblasts are more common while in the Chronic form (CML), myelocytes are the dominant cells.
86. A mutation has occurred in a cell in the first exon of the structural gene. The number of nucleotide pairs changed from 290 to 250. Name this type of mutation:

Explanation

Deletion- removal of a chromosome.

Aneuploidy- abnormal set of chromosomes 

Inversion- rearrangement of chromosomes

 

Translocation- transfer of an arm of a chromosome to another.

87. The dorsal root of the spinal nerve of a test animal was severed. What changes will occur in the innervation area?

Explanation

 

The dorsal root of the spinal cord is responsible for transmitting sensory information. The sensory fibres enter the spinal cord through the dorsal root. The ventral root is responsible for motor actions. The motor fibres exit the spinal cord through the ventral root.

88. Due to destruction of certain structures of the brainstem a test animal has lost its orientation reflexes in response to strong light stimuli. What structures were destroyed?

Explanation

The quadrigeminal plate is a part of the brain responsible for orientational reflexes ( light and sound signals). It consists of the Inferior colliculi (posterior quadrigeminal bodies) -for sound and the superior colliculi (anterior quadrigeminal bodies)- for light signals. The substantia nigra produces dopamine and the red nucleus controls tone of flexors and extensors.
89. A toxin neutralized with 0.4% formaldehyde under 37-40oC for 4 weeks is used for vaccination. This preparation was first used by Gaston Ramon for diphtheria prevention. Name this preparation:

Explanation

Anatoxin also known as toxoids are toxins which toxicity have been suppressed after being treated by a chemical. Its immunogenic properties are still preserved as they are used to generate an immune response against harmful toxins eg, diphtheria

 

Antitoxic serum are artificial antibodies used in the treatment of diphtheria. Inactivated vaccines are used in cases such as influenza, typhoid.

90. 24 hours after an appendectomy the patient’s blood test shows neutrophilic leukocytosis with a regenerative shift. What is the most likely mechanism of absolute leukocytosis development in the patient’s peripheral blood?

Explanation

 

In neutrophilic leukocytosis with regenerative shift, immature, blast cells are more. This indicates an increase in leukopoiesis (generation of new white blood cells).

91. The right leg of a 40-year-old woman measured at the shin level is by 2 cm smaller in the diameter than the left leg. Anklejerk (Achilles) and knee-jerk reflexes are absent on the right. What is the most likely mechanism of hyporeflexia development during peripheral paralysis?

Explanation

A reflex arc is a neural path that controls a reflex action. It consists of receptors, sensory neurons, motor neurons and effector organs. Absence of reflex on the right leg occured due to a defect in the conduction pathway ie, receptors and neurons.

 
92. The right leg of a 40-year-old woman measured at the shin level is by 2 cm smaller in the diameter than the left leg. Anklejerk (Achilles) and knee-jerk reflexes are absent on the right. What is the most likely mechanism of hyporeflexia development during peripheral paralysis?

Explanation

A reflex arc is a neural path that controls a reflex action. It consists of receptors, sensory neurons, motor neurons and effector organs. Absence of reflex on the right leg occured due to a defect in the conduction pathway ie, receptors and neurons.

 
93. A 38-year-old man, who has been suffering from systemic lupus erythematosus for 3 years, developed diffuse renal lesions accompanied by massive edemas, marked proteinuria, hyperlipidemia, and dysproteinemia. What is the most likely mechanism of proteinuria develoment in this case?

Explanation

 

Systemic lupus erythematosus ( characterised by malar/ butterfly rash) is an autoimmune disease classified under type 3 hypersensitivity ( immune complex). Triggered by environmental issues, the body quickly produces antibodies against these antigens and its own tissues forming an antigen-antibody complex. These complexes are deposited in the kidneys which leads to an inflammatory reaction in the glomerulus ( glomerulonephritis) thereby, destroying the nephron.

94. A 59-year-old man, a business manager, developed intense burning retrosternal pain that irradiates to the left arm. The pain occurred in the evening after the tax audit. 15 minutes later the patient’s condition normalized. What mechanism of angina pectoris development is leading in this patient?

Explanation

 

Intense burning retrosternal pain that radiates to the left arm indicates angina pectoris. It is caused by vasospasm (constriction) of coronary vessels. In the above question, patient was in a tense situation due to the tax audit. This led to the release of catecholamines (adrenaline, noradrenaline) which are potent vasoconstrictors.

95. A 25-year-old woman at her third pregnancy with impending miscarriage was brought to the hospital. What combination of Rh-factor of the mother and the fetus can be the cause of this condition?

Explanation

 

Rhesus incompatibility is observed when the mother possesses an Rh -ve blood group and the developing child has an Rh +ve blood group (most likely gotten from the father).

96. A specimen of a 10-day-old human embryo shows two interconnected sacs (amniotic and yolk sacs). Name the structure located in the place where these two sacs connect:

Explanation

Found between the amniotic and yolk sacs is a flattened inner cell mass from which differentiation takes place.
97. A woman with the III (В), Rh (-) blood group gave birth to a child with the II (А) blood group. The child is diagnosed with hemolytic disease of newborn caused by rhesus incompatibility. What blood group and Rh does the father have?

Explanation

 

Rhesus incompatibility is observed when the mother possesses an Rh -ve blood group and the developing child has an Rh +ve blood group (most likely gotten from the father). The child’s II (A) blood group is also inherited from the father.

98. A 40-year-old woman suffers from Cushing’s disease - steroid diabetes. On biochemical examination she has hyperglycemia and hypochloremia. What process activates in the first place in such patients?

Explanation

Cushing’s disease refers to a hyper function of the adrenal cortex. N/B The adrenal cortex is made of 3 zones namely;

Zona Glomerulosa: Mineralocorticoids eg. aldosterone

Zona Fasciculata; Glucocorticoids

Zona Reticularis: Androgens

 

There is an increased secretion of glucocorticoids, cortisol which elevate blood glucose level by activating gluconeogenesis.

99. A patient with a chemical burn has developed esophageal stenosis. The patient presents with acute weight loss due to problematic food intake. Blood test: erythrocytes - 3.0 · 1012/L, Hb- 106 g/L, total protein - 57 g/L. What type of starvation does this patient suffer from?

Explanation

Starvation is simply the deficiency of food leading to absence of energy. In incomplete starvation, there is an intake of water. Complete starvation has to do with total absence of food and water. In esophageal stenosis, there is a narrowing of the esophagus making swallowing difficult but water and liquid products are still able to pass through.

 
100. Autopsy of the body of a man, who for a long time had been working at the factory with high content of silicon dioxide in the air, revealed enlarged dense lungs with numerous round and oval sclerotic foci. The foci were miliary and larger in size and colored gray or gray-black. What is the most likely diagnosis?

Explanation

Silicosis is a lung disease that occurs due to inhalation of dust containing silica. Nodular silicosis is characterised by numerous round or oval sclerotic foci. The diffuse form covers a larger area of the lungs.

 
101. Autopsy of the body of a 50-yearold man, who had been suffering from tuberculosis and died of cardiopulmonary decompensation, shows lobar affection of the lungs: the upper lobe of the right lung is enlarged, dense, yellow on section, crumbling, with pleural fibrin deposition. What type of secondary tuberculosis can be characterized by this pathology?

Explanation

Caseous pneumonia occurs due to progression of secondary tuberculosis. Characterised by the deposition of fibrin, enlarged, dense and yellow in section.
102. A woman has come to the hospital with complaints of general weakness, dizziness, and dyspnea. Resently she has been taking levomycetin (chloramphenicol) for prevention of enteric infection. Blood test: erythrocytes - 1.9 · 1012/L, hemoglobin - 58 g/L, color index - 0.9, leukocytes - 2.2 G/L, reticulocytes - 0.3%. What type of anemia is it indicative of?

Explanation

Hypoplastic anemia is a disorder in which the bone marrow produces low amounts of RBC. The intake of Levomycetin ( chloramphenicol) has an adverse effect on the bone marrow ie, it suppresses the bone marrow leading to a decrease in its production rate causing hypoplastic anemia. In iron deficiency anemia, the color index will be low Norm; 0.85-1.05. In aplastic anemia, the bone marrow completely stop production- reticulocyte ( immature Rbc ) will be totally absent.
103. Preoperative examination revealed prothrombin deficiency in the blood of the patient. What drug should be preliminarily prescribed to mitigate blood loss in the patient during the surgery?

Explanation

Menadione or vicasol is a vitamin K supplement indicated in bleeding/hemorrhage.
104. A patient with streptococcal infection of the gingiva was prescribed a drug with β-lactam ring in its structure. What drug of those listed below belongs to this pharmacological group?

Explanation

Beta lactam is a group of antibiotics that destroy bacterial cell wall. They do this by inhibiting Transpeptidase enzyme ( responsible for cross linking of fibres of bacterial cell wall). To this group belong Penicillins, Cephalosporins, Monobactam and carbapenems.
105. A microslide of the skin sample taken from the finger of a child shows that epidermis is insufficiently developed. What germ layer was damaged in the process of embryo development?

Explanation

Ectoderm- brain, epidermis of skin and pigment cells (melanocytes).

Mesoderm- Dermis, smooth and skeletal muscles

Endoderm- GIT

 
106. Neutralization of xenobiotics and active endogenous metabolites often occurs via introduction of an oxygen atom into the substrate molecule. What process occurs as the result?

Explanation

 

Hydroxylation is a chemical process in which a hydroxyl group -OH is added, it is facilitated by the enzyme hydroxylases. Decarboxylation is the removal of a carboxyl group thereby releasing CO2. Phosphorylation is an addition of phosphoryl group. Deamination is the removal of an amine group.

107. In the process of an experiment, vascular resistance to the blood flow was measured in the different areas of circulatory system. The highest resistance was detected in the:

Explanation

Peripheral or vascular resistance is the resistance of the arteries to blood flow. It is maximum at the level of the arterioles.
108. A 34-year-old man died in a comatose state. According to his family after a business trip to an African country he developed periodical jaundice attacks. Autopsy shows the following: dense enlarged spleen with slate-black pulp; enlarged plethoric liver, gray-black on section; cerebral gray matter is brown-gray; cerebral white matter contains numerous small hemorrhages. What infectious disease can be suspected?

Explanation

Hemomelanin is a dark pigment produced by malarial parasites from hemoglobin. It is taken up by monocytes in the blood and subsequently deposited in the liver and spleen giving it that dark colour. Meningococcemia is characterised by hemorrhages in the meninges caused by infection from Neisseria meningitidis
109. What diagnostic method should be used in industry to test the raw leather for presence of B. antracis?

Explanation

Bacillus anthracis is a gram positive, spore forming, rod shaped bacteria that causes Anthrax. For diagnosis of anthrax, Ascoli’s thermo precipitation test is used. Burri Gin’s method is used for encapsulated organisms ie, organisms that have capsules.

 
110. A 46-year-old patient suffering from ulcer disease of the stomach is diagnosed with rheumatoid arthritis. What antiinflammatory drug would be the most advisable in this case?

Explanation

 

Celecoxib is a selective COX-2 non steroidal anti-inflammatory drug used for the treatment of diseases like rheumatoid arthritis, ankylosing spondylitis etc. Prednisolone is a glucocorticoid, has anti-inflammatory, immunosuppressive effects.

111. A patient with myocardial infarction has acute heart failure. Among the drugs that increase the force of heart contractions the least dangerous in this case will be:

Explanation

 

Dobutamne is a cardiotonic, a beta 1 agonist with positive inotropic effect (increases force and speed of contraction). It is proposed as a cardiotonic after myocardial infarction or open heart surgery.

 
112. During intensive physical exertion, one of the energy sources for the working muscles is glucose produced as the result of gluconeogenesis. This process is the most intensive in the following organ:

Explanation

Gluconeogenesis is a metabolic process in which glucose is synthesized from non carbohydrate components eg, lipids, protein. It is more intense in the liver and lesser in the cortex of kidneys.
113. During inflienza epidemic, morbidity in the schoolchildren, who did not participate in sports, was 40%, while in the schoolchildren, who engaged in regular physical acivities, morbidity did not exceed 20%. What adaptation mechanism ensured low morbidity in the physically active schoolchildren?

Explanation

 

Cross adaptation is seen when an exposure to a particular stressor/stimulus ( in this case exercise) provides protection/tolerance from another stimulus.

114. Monoamine oxidase inhibitors are widely used as psychopharmacological drugs. They change the level of nearly all neurotransmitters in synapses, with the following neurotransmitter being the exception:

Explanation

 

Monoamine oxidase inhibitors are drugs used in patients with depression. They increase the secretion of biogenic amines ( by decarboxylation) ie, serotonin, dopamine, adrenaline and noradrenaline. It has no effect on the neurotransmitter acetylcholine.

115. A 17-year-old girl suffers from periodical palpitations that last several minutes. Her heart rate is 200/min., rhythmic. What heart rhythm disorder developed in this patient?

Explanation

Tachycardia simply means a fast heart rate (faster than normal- 60-100beats/min). Paroxysmal means a sudden attack or recurrence of symptoms. So in paroxysmal tachycardia, the tachycardia attack begins and ends suddenly (it lasts for several minutes).
116. Kidney X-ray image obtained by means of pyelography shows that the minor calyces converge to form 2 major calyces that adjoin to the renal pelvis, from which the ureter exits. What type of urinary tract is it?

Explanation

In a mature urinary tract, 2 minor calyces merge to form a major calyx which runs into the renal pelvis and continues as the ureter.  In embryonal urinary tract, major calyces are absent.
117. A patient has been brought into a surgical ward with an incised wound of the anterior surface of the shoulder in its lower one-third. Flexing function was disturbed in the shoulder and elbow joints, which is caused by the damage to the:

Explanation

 

The biceps muscles of the shoulder are located at the anterior surface in the lower one-third and is responsible for flexion. The Triceps muscles of the shoulder are located on the posterior surface in the lower one-third and are responsible for Extension.

118. A patient suffers from acute cardiopulmonary failure with pulmonary edema. What diuretic should be prescribed in the given case?

Explanation

Furosemide is a loop diuretic indicated in cerebral, pulmonary edema for forced diuresis. Spironolactone is a potassium sparing diuretic (keeps potassium in the system). Hydrochlorothiazide is a thiazide diuretic. Diuretics are substances that increase the excretion of water and salt from the body.

 
119. Ketosis develops in the patients with diabetes mellitus, as the result of activation of fatty acids oxidation processes. What acidbase imbalance can result from accumulation of excessive ketone bodies in the blood?

Explanation

Acidosis is an increase in H+ ions in the blood. Metabolic acidosis is seen in cases of diabetes mellitus due to the accumulation of ketone bodies. Respiratory acidosis is observed in hyperventilation. Alkalosis is an increase of bicarbonates in the blood. Respiratory alkalosis is seen in hypercapnia ( increase of CO2 in the blood).Metabolic alkalosis is seen in vomiting.
120. A 5-year-old child is diagnosed with Bruton syndrome (X-linked agammaglobulinemia) that manifests itself as severe clinical course of bacterial infections and absence of B lymphocytes and plasma cells. What changes of immunoglobulin content can be observed in blood serum of the child with immunodefi- ciency?

Explanation

Bruton’s agammaglobulinemia is an X-linked immunodeficiency disease that occurs due to the mutation in the gene that codes Bruton tyrosine kinase enzyme (BTK). This enzyme helps B lymphocytes in maturing. Since B cells are responsible for the production of immunoglobulins, a reduction in antibody levels are observed especially IgA and IgM levels.
121. During the skill-building session in microbiology the students need to stain the prepared and fixed sputum smears obtained form a tuberculosis patient. What staining technique should be used in this case?

Explanation

 

Tuberculosis is caused by Mycobacterium tuberculosis. An acid-fast bacilli. Ziehl neelsen stain is indicated for its staining procedure.

122. A woman with hypophyseal diabetes insipidus developed a water-mineral imbalance. What type of water-mineral imbalance develops in such cases?

Explanation

 

Diabetes insipidus is a disease characterised by excretion of large amounts of diluted urine (dehydration) accompanied by a feeling of thirst. The loss of water also makes the blood more concentrated than normal ie, high concentration of salts, glucose and other substances. This is called hyperosmolarity.

123. First-year schoolchildren have received tuberculin skin test (Mantoux test) at the school nurse’s office. The purpose of this test was:

Explanation

Before the application of BCG vaccine ( vaccination against tuberculosis), A skin test (mantoux test) is done. In this test, tuberculin is injected intradermally and the injected area is observed. A person who has been exposed to the bacteria is expected to mount an immune response in the skin containing the bacterial proteins. The reaction is read by measuring the diameter of induration. This test helps in determining individuals in need of BCG vaccination.

 
124. In an experiment it is necessary to assess neuromotor and muscle excitability. What value shoud be measured to make the assessment?

Explanation

 

A sensory threshold is the level of strength a stimulus must reach to be detected. Resting membrane potential is the voltage around the cell membrane when the cell is in a quiet/resting state.

125. Acute renal impairment caused death of a patient with hemorrhage. Autopsy revealed enlarged kidneys with broad pale pink cortical layer expressively demarcated from dark red renal pyramids. Macroscopic examination revealed lack of epithelial nuclei of the convoluted tubules, tubulorrhexis, phlebostasis. The cell nuclei of the choroid glomus and straight tubules were present. What pathology is it?

Explanation

Necronephrosis is a condition characterised by the death of epithelial cells in the tubules of the nephron.lack of epithelial nuclei of the convoluted tubules, tubulorrhexis, phlebostasis all indicate the death of breakdown of tubules. Glomerulonephritis involves the inflammation of the filtration apparatus.
126. Vestibular receptors of semicircular canals of a test animal have been destroyed. What reflexes will disappear as a result?

Explanation

 

The three semicircular canals are found in the innermost part of the middle ear. They are part of the bony labyrinth. At each end, they have an ampulla. These canals detect angular acceleration of the head. The utricle and saccule respond to linear acceleration.

127. A boxer had sustained a blow to the left parotid area, after which he developed paralysis of the facial muscles on this side. What nerve had been damaged?

Explanation

The facial nerve innervates the facial muscles and is responsible for facial expressions. The opthalmic, maxillary and mandibular nerves are branches of the trigeminal nerve.

 
128. A patient has an open facial wound with undermined edges; tissue necrosis with gradually developing partial gangrene that nearly reaches the bone tissue is observed. On close examination the wound contains live larvae. The patient is diagnosed with tissue myiasis. What Diptera larvae are the causative agents of this disease?

Explanation

Wohlfahrtia magnifica (wohlfahrt fly) is the causative agent of myiasis (Myiasis is the parasitic infestation of the body of a live animal by fly larvae (maggots) that grow inside the host while feeding on its tissue.

 
129. A hereditary disease - homocystinuria - is caused by disturbed transformation of homocysteine into methionine. Accumulated homocysteine forms its dimer (homocystine) that can be found in urine. What vitamin preparation can decrease homocysteine production?

Explanation

 

Vitamins such as B12, B9 and B6 help reduce homocysteine levels.Homocysteine is an amino acid made from a common dietary amino acid, methionine, that inflicts damage to the inner arterial lining (endothelium) and contributes to many diseases such as migraines, strokes, cardiovascular related diseases etc.

130. During hypersensitivity skin test a patient received an allergen subcutaneously, after which the patient developed skin redness, edema, and pain due to histamine action. This biogenic amine is produced as the result of the following transformation of histidine amino acid:

Explanation

 Histamine (a biogenic amine) is produced from the decarboxylation of histidine protein. Decarboxylation refers to the removal of a carboxyl group thereby releasing CO2.

 
131. During the fight a man has received a strong blow to the upper anterior abdominal wall, which resulted in the cardiac arrest. What mechanism has led to the cardiac arrest in this case?

Explanation

A cardiac arrest is simply when your heart suddenly stops working (pumping blood). On activation of the parasympathetic nervous system, heart activities are decreased. Unconditioned reflexes refers to inborn reflexes ie, reflexes not learned eg. accommodation reflex. Conditioned reflexes are learned over time ( through training).
132. A woman has lost a lot of blood during the childbirth. Her blood group needs to be determined. Erythrocyte agglutination occurred with standard serums 0 (I) and А (II) and did not occur with standard serum B (III). What blood group does this woman have?

Explanation

Agglutination not occuring in standard serum B(III) indicates the blood group belongs to B(III).Blood is often grouped according to the ABO blood typing system. The 4 major blood types are;Type O(I),Type A(II),Type B(III),Type AB(IV).

  •  type A blood can only receive types A and O blood.

  •  type B blood,can only receive types B and O blood.

  • type AB blood,can receive types A, B, AB, and O blood. -universal recipient

  •  type O blood,can only receive type O blood.

  •  Rh+, you can receive Rh+ or Rh- blood.

  •  Rh-, you can only receive Rh- blood.

 

Type O blood can be given to anyone with any blood type. That is why people with type O blood are called universal blood donors.

133. A 60-year-old man diagnosed with chronic heart failure was brought to the hospital. After a long course of treatment the patient developed signs of intoxication: dyspnea, extrasystole, nausea, and disturbed perception of colors. What medicine has caused such side-effects?

Explanation

 

Digoxin is a cardiac glycoside used in the treatment of chronic heart failure. Side effects include dyspnea, extrasystole etc. Nitroglycerin is a vasodilator used in the treatment of angina. Drotaverine is an antispasmodic drug. Anaprillin is a non selective Beta blocker.

134. A patient with exacerbated peptic ulcer disease of the stomach has made an appointment with the doctor. What type of drugs should be included in the complex therapy of this patient?

Explanation

H2 blockers should be included in complex therapy for ulcer treatment eg, Famotidine, ranitidine, cimetidine etc Other groups include Proton pump inhibitors eg, omeprazole and mucoprotectors.
135. Autopsy of the body revealed waxy degeneration of the rectus abdominis muscles. In the terminal segment of the small intestine there are ulcers 3-5 cm in diameter. The ulcer walls are covered in a crumbling grayish-white substance. The ulcer edges are markedly raised above the mucosa. Widal test is positive. Make the diagnosis:

Explanation

Widal agglutination test is a diagnostic method used for the diagnosis of typhoid fever ( enteric fever) which is marked by the formation of multiple ulcers, hemorrhages covered with grayish substances (fibrin) in the intestine.

 
136. Genealogical analysis of a child with myotonic dystrophy determined that this disease is present in every generation, equally presented by the relatives of both genders, the risk of inheriting this disease is equal no matter which parent is affected. If one of the parents is heterozygous for this disease and the other parent is healthy, the risk of them giving birth to a sick child is 50%. What type of disease inheritance is it?

Explanation

 

An autosomal dominant inheritance is one that is present in all generations, affects both genders and doesn't need both parents to be affected for its expression. Y-linked disorders affects only males and is passed from father to son. X-linked disorders are passed from father to daughter or mother to any child.

137. A 30-year-old man has undergone surgical removal of a thyroid tumor. Histologically the tumor is made up of papillary structures varying in size that emerge from the inner surface of dilated cystic follicles and are covered with atypical epithelium. What is the most likely diagnosis?

Explanation

 

Papillary carcinoma  is one of the four main types of malignant tumours derived from thyroid follicle cell (the others include; follicular, medullary and anaplastic carcinoma). Papillary form is characterised by  papillary structures varying in size, it metastasizes via lymphatics to nodes in the neck. Follicular form possess follicular structures and metastasizes via bloodstream to bone tissues. Medullary form originates from parafollicular cells  or C cells.

138. A young woman, a foreign student from Tehran, has made an appointment with the urologist. She complains of the sensation of heaviness in her lower abdomen and a small amount of blood being excreted with urine at the end of each urination. Microscopy of urine detects the presence of parasite eggs, approximately 140х70 micron in size, with a terminal spike. What diagnosis can be made by the infectious diseases specialist?

Explanation

 

Schistosomiasis are gotten from parasitic flatworms called schistosomes (blood flukes). On examination, these parasitic eggs contain terminal spikes.N/B Snails are the intermediate host, infection is gotten from the consumption of contaminated water. In Opisthorchiasis, the parasitic eggs are small, operculate. Fascioliasis is gotten from Fasciola hepatica.

139. Various biological preparations can be used for poliomyelitis prevention. What drug induces the type of local intestinal mucosal immunity that lasts the longest?

Explanation

 

Poliomyelitis is an infectious disease caused by the polio virus. For prevention and production of immunity against this virus, a live/attenuated vaccine is given orally.

140. Histological analysis of a removed skin tumor shows clusters and bands composed of atypical stratified squamous epithelium cells that penetrate into the underlying tissue. What preliminary diagnosis can be made?

Explanation

 

Non keratinizing squamous cell carcinoma is composed of atypical squamous cells that penetrates underlying tissues.  Papilloma is a benign tumor( no cellular atypia ) that originates from skin or mucous membrane, presents with a papillary surface.Adenomas are benign tumors starting in the epithelial tissue of a gland or gland-like structure.

141. Autopsy of a patient, who died of bilateral bronchopneumonia, shows in the left lung lower lobe a cavity 5 cm in diameter, filled with yellowish-white liquid. What complication of the patient’s pneumonia had developed?

Explanation

An abscess is a cavity containing purulent exudates. Sequestrum is a fragment of dead tissues that can't be autolysed (destroyed) located among alive tissues.

 
142. A patient with parkinsonism was prescribed levodopa, which led to rapid improvement of the patient’s condition. What is the mechanism of action of this drug?

Explanation

Parkinson’s disease is a disorder of the nervous system  caused by decreased synthesis in dopamine ( a neurotransmitter). Dopamine is produced from the substantia nigra ( black substance) found in the brain. Symptoms include; tremors, stiffness in movements etc. Levodopa is used in treatment, it stimulates dopamine synthesis.

 
143. A sick child presents with high content of phenyl pyruvate in urine (normally it is practically absent). Blood phenylalanine level is 350 mg/L (norm - 15 mg/L). What disease are these symptoms characteristic of?

Explanation

Phenylketonuria is a disorder related to the disturbed breakdown of the amino acid phenylalanine. It occurs due to the deficiency of the enzyme phenylalanine hydroxylase. Patients with this disorder have yellow-green/ lemon coloured urine. In alkaptonuria, Homogentisic acid is accumulated; urine is usually coloured black. In gout, uric acid is accumulated.
144. An oncology patient is to undergo a surgery on the descending colon. Name the main source of blood supply to this organ:

Explanation

 

Branches of the superior mesenteric artery supply the small intestine,caecum, ascending colon and a part of the transverse colon.Branches of the inferior mesenteric artery supply the remaining half of the transverse colon, the descending colon, sigmoid down to the rectum.

145. A patient presents with enlarged cervical lymph nodes. Other lymph nodes and internal organs are without changes. Peripheral blood test results are normal. Histological examination of biopsy material taken from the cervical lymph node shows smoothed-out pattern, absent follicles, homogeneous cell composition represented by lymphoblasts. The cells penetrate into the lymph node capsule. What diagnosis can be made?

Explanation

Lymphoblastic leukemia is a form of blood cancer in which the bone marrow produces a lot of lymphoblasts. Burkitt’s lymphoma is a B cell type of lymphoma typical in people with Malaria and Epstein Barr virus.
146. When studying the pulmonary ventilation values, the reduction of forced expiratory volume has been detected. What is the likely cause of this phenomenon?

Explanation

Ventilatory dysregulation - Morphine poisoning , Tetanus

      Ventilatory obstruction - Bronchial asthma, COPD

      Ventilatory restriction - silicosis, anthracosis etc.  

In obstructive diseases, the forced expiratory volume(FEV) is reduced.

 
147. A man is a carrier of HIV that is an RNA virus. The cells of this patient synthesize viral DNA. This process is based on:

Explanation

When HIV infects a cell, reverse transcriptase copies the viral single stranded RNA genome into a double-stranded viral DNA. This process is called reverse transcription.

Translation is a process that occurs in ribosomes present in the endoplasmic reticulum in which proteins are synthesized from RNA molecules. It occurs after replication and has three phases: initiation, elongation and termination.

 

In Transcription, DNA is copied into RNA by the help of RNA polymerase.

148. Sanitary assessment of a pond, where the children from a recreation summer camp take their swims, detected there oval cysts 50-60 micron in diameter, with 2 nuclei visible in their cytoplasm (macronucleus and micronucleus). What protozoa do these cysts belong to?

Explanation

Balantidum is the causative agent of balantidiasis. It possesses 2 nuclei (macro and micro) in the form of an oval cyst. Lamblia possesses 2 nuclei and 4 flagellas. Amoeba is unicellular, shapeless and has pseudopods.
149. A hospital has received a 24-year-old man, who had received a penetrating wound to the eye, which has caused the vitreous body to run out. As the result of this, retinal detachment occurred. What retinal layer was tightly adherent to the vascular tunic of the eye and did not detach?

Explanation

 The retinal pigment epithelium are specialised cells located between the neurosensory cells (rods and cones) and the choroid capillaries (the vascular tunic of the eye). One major function is light absorption. Rods are for twilight vision while cones are for coloured vision.

 
150. A newborn presents with weak suckling, frequent vomiting, and hypotonia. Blood and urine citrulline are very high. What metabolic process is disturbed?

Explanation

 

In ornithine/urea cycle, Citrulline is converted into aspartate.

151. Stool test detects in the patients feces a large amount of undigested fats.This patient is the most likely to have disturbed secretion of the following enzymes:

Explanation

 

Pancreatic lipase also known as triglyceride lipase is an enzyme responsible for the breakdown(hydrolysis) of triglycerides into monoglycerides and free fatty acids. Bile lipase helps in the digestion of fats.

152. A 19-year-old young man was examined in the nephrology clinic. High calcium was detected in his secondary urine. What hormone is likely to cause such change, if it is produced in excess?

Explanation

Chronic mineralocorticoid excess states in humans, is known to cause a fall in plasma concentrations of ionized Ca2+ and Mg2+ by increased urinary and fecal excretion of these ions. N/B The adrenal cortex is made of 3 zones namely;

Zona Glomerulosa: Mineralocorticoids eg. aldosterone

Zona Fasciculata; Glucocorticoids

Zona Reticularis: Androgens

 
153. A young woman presents with a tumor along the auditory nerve. The tumor is node-shaped, 3 cm in diameter, soft and elastic, pink-white colored, and has homogeneous structure. Microscopically the tumor contains bundles of cells with oval nuclei. These cellular fibrous bundles form regular structures made up of parallel rows of regularly oriented cells arranged in the form of a palisade with acellular homogeneous area in between (Verocay bodies). Name this type of tumor:

Explanation

 

Presence of Verocay bodies indicates a neurinoma which is a benign tumour that grows on the vestibulocochlear nerve (nerve for hearing and balancing) in the internal ear.

154. A patient has undergone surgical removal of the pylorus. Decreased secretion of the following hormone can be expected:

Explanation

Gastrin is a peptide hormone secreted by G cells present in the pyloric antrum of the stomach. It stimulates the release of HCL from parietal cells.
155. An electron micrograph shows a small vessel with endothelial layer but without basement membrane and pericytes; anchoring fibrils are present. Name this vessel:

Explanation

Pericytes are cells in microcirculation that surround the endothelial cells of venules and capillaries; they have contractile function. Lymphatic capillaries have no basement membrane (basal lamina) and pericytes.
156. One of the causes of pernicious anemia is disturbed synthesis of transcorrin - Castle’s intrinsic factor - by the parietal cells of the stomach. What substance is called Castle’s extrinsic factor?

Explanation

 

Intrinsic factor of Castle refers to glycoprotein produced by parietal cells in the gastric walls that enable the absorption of vitamin B12. Extrinsic factor refers to Vitamin B12- cobalamin.

157. A patient with contact dermatitis needs to be prescribed an antihistamine drug without somnolescent effect. Select this drug from the list:

Explanation

Somnolescent effect refers to drowsiness, sleepiness. Loratadine is a H1-blocker (antihistamine) indicated in rhinitis, contact dermatitis etc, it doesn’t cause drowsiness or sleepiness after intake. Benadryl and suprastine are also antihistamines but have somnolescent effect.
158. Congenital pyruvate carboxylase defi- ciency causes physical and mental retardation in children and leads to early death. It is characterized by lactic acidemia, lactaciduria, and a number of metabolic disorders. Among others, inhibition of the following occurs:

Explanation

During gluconeogenesis, pyruvate carboxylase is involved in the synthesis of phosphoenolpyruvate (PEP) from pyruvate.Pyruvate carboxylase deficiency is an inherited disorder that causes lactic acid and other potentially toxic compounds to accumulate in the blood.This enzyme is active in mitochondria, which are the energy-producing centers within cells.Aside from the role of PC in gluconeogenesis, PC serves as an intermediate for the tricarboxylic acid cycle (essential to provide oxaloacetate) It is involved in several important cellular functions, including the generation of glucose.
159. A woman with enteritis accompanied by severe diarrhea presents with loss of water in the extracellular space, increased water content in the cells, and decreasing blood osmolarity. Name this type of waterelectrolyte imbalance:

Explanation

Loss of water in the extracellular space indicates hypohydration; decreased blood osmolarity implies a hypoosmolar state. Hypoosmolar hypohydration is seen in diarrhea.
160. To stimulate the labor activity a parturient woman was prescribed a drug - a posterior pituitary hormone that does not affect the blood pressure. As the pregnancy progresses, the sensitivity to this hormone increases. Name the prescribed drug:

Explanation

 

Oxytocin is a hormone produced in the posterior pituitary gland. It is responsible for muscle contraction during labour(childbirth). Dinoprostone is a prostaglandin E2, it is used in pregnant women to relax the cervix muscles in preparation for labour. Dinoprost is a prostaglandin F2 alpha, it is used to induce arbortion in the 2nd trimester of pregnancy.

161. Cell membrane rest potential changed from -85 to -90 mV. It can be caused by activation of the following cell membrane channels:

Explanation

 

A change of membrane potential from -85 to -90 mV indicates repolarization. Depolarization- sodium influx to the cell. Repolarization- Potassium efflux.

162. Ionizing radiation or vitamin E deficiency affect the cell by increasing lysosome membrane permeability. What are the possible consequences of this pathology?

Explanation

Vitamin E (alpha tocopherol) is an antioxidant - responsible for killing off Reactive oxygen species (bad oxygen). These bad oxygen destroy lysosomes thereby releasing the contents ie, hydrolytic enzymes. These enzymes lead to the partial or complete destruction of cells.
163. A patient is diagnosed with glucocerebroside lipidosis (Gaucher’s disease) that manifests as splenomegaly, liver enlargement, affected bone tissue, and neuropathies. What enzyme of complex lipid catabolism is deficient, thus causing this disease?

Explanation

 

Gaucher’s disease (glucocerebrosidase lipoidosis) is a genetic disorder in which glucocerebroside is accumulated in cells (monocytes, macrophages). It occurs as a result of the deficiency of the enzyme glucocerebrosidase. Neimann pick’s disease is a lysosomal storage disorder in which the body is not able to metabolise fats, sphingomyelin is accumulated in the lysosomes. It occurs due to the deficiency of the enzyme sphingomyelinase.

164. A 3-year-old girl with mental retardation has been diagnosed with sphingomyelin lipidosis (Niemann-Pick disease). In this condition the synthesis of the following substance is disturbed:

Explanation

 

Neimann pick’s disease is a lysosomal storage disorder in which the body is not able to metabolise fats, sphingomyelin is accumulated in the lysosomes. It occurs due to the deficiency of the enzyme sphingomyelinase.

165. A traumatology unit has received a patient with a wrist trauma and a clinical presentation of the damage to the nerve that passes through the carpal tunnel. Name this nerve:

Explanation

166. A patient, who has been taking β- adrenergic blockers, developed a bronchial spasm. What group of bronchodilators should be chosen to stop the bronchial spasm?

Explanation

 

As a result of the action of the B-adrenergic blockers, bronchial spasm occurred. Myotropic antispasmodic also known as spasmolytics should be prescribed to relieve such symptoms examples include papaverine, tizanidine.

167. A man has an inguinal hernia. The hernial sac exits through the following opening:

Explanation

Inguinal Hernia simply means the protrusion/bulging out of abdominal content from the inguinal canal. The annulus superficialis canalis inguinalis ie, superficial/external inguinal ring is a triangular opening that forms the exit of the inguinal canal.
168. After a nephrectomy the patient developed enteroparesis. What cholinergic agent with anticholinesterase action should be prescribed in this case?

Explanation

Proserin is a drug with anticholinesterase action indicated in intestinal peristalsis.

169. An obstetrician-gynecologist measures pelvis size of a pregnant woman. A caliper was used to measure the distance between the two iliac crests. What measurement of large pelvis was made?

Explanation

Distantia cristarum is the distance between the farthest points of the two Iliac crest ∼29cm in females. Distantia throcantherica is the distance between the femurs. Distantia spinarum is the distance between anterior supra iliac spines.
170. Parenchyma of an organ is composed of pseudounipolar neurons localized under the capsule of connective tissue. Central place belongs to nerve fibers. Name this organ:

Explanation

Pseudounipolar neuron is a type of neuron in which the axon divides into two branches. They are found in the spinal ganglion. Bipolar neurons are found in the cells of the retina.
171. Genetic recombination is achieved via several mechanisms. One such mechanism is crossingover. It occurs at the following stage of prophase in the first meiotic division:

Explanation

 

Prophase 1 is essentially the crossing over and recombination of genetic material between non sister chromatids - this results in the genetically unidentical, haploid daughter chromatid cells. In meiosis, it has 5stages ie, Leptotene, Zygotene, Pachytene, Diplotene and Diakinesis.Leptotene is the first of five stages of Prophase 1 and consists of the condensing of the already replicated chromosomes.Zygotene is the sub-stage where synapsis between homologous chromosomes begins. Crossing over occurs in pachytene phase. In diplotene, the two homologous chromosomes begin to migrate apart .Diakinesis is the final step of Prophase 1 and is the termination of the condensing of the chromosomes. The chromosomes are at their most condensed form during diakinesis.

172. ECG of the patient shows increased duration of the QRS complex. What is the most likely cause?

Explanation

 

P-wave indicates atrial excitation/depolarization. QRS indicates ventricular depolarization/excitation. T-wave indicates ventricular repolarization/relaxation.

173. A 50-year-old inpatient during examination presents with glucosuria and blood glucose of 3.0 mmol/L, which are the most likely to be caused by:

Explanation

Low blood glucose level (norm 3.5-5.5 mmol/L) and high glucose in urine indicates the pathology lies in kidney ie, with filtration and reabsorption. N/B in physiological state, the kidney does not filter out glucose unless it surpasses it renal threshold.
174. A 40-year-old woman was diagnosed with glomerulonephritis based on her clinical symptoms and the results of urine analysis. Anamnesis states chronic tonsillitis. What microorganisms are the most likely cause for her kidney damage?

Explanation

 

Glomerulonephritis is a term used for a group of diseases involving the inflammation of the glomeruli. In the post infectious form of GN, it occurs after the infection of streptococcus pyogenes typically after a pharyngeal infenfection (1-3 weeks after).

175. A 7-year-old child presents with marked signs of hemolytic anemia. Biochemical analysis of erythrocytes determined low concentration of NADPH and reduced glutathione. What enzyme is deficient in this case leading to the biochemical changes and their clinical manifestations?

Explanation

Erythrocytes possess no nucleus, the main metabolic pathway in these cells is the pentose phosphate pathway in which NADPH is produced in the process.Glucose 6 phosphate dehydrogenase is the enzyme that catalyzes the reaction in which NADPH is produced from NADP. In lipid peroxidation, reactive oxygen species are activated (bad oxygen that kills cells). NADPH serves as the ultimate donor of reductive power for the large majority of ROS-detoxifying enzymes thereby counteracting lipid peroxidation.
176. When examining a biopsy material obtained from the thyroid gland, the pathologist discovered lymphocyte infiltration of the thyroid tissues and destruction of the parenchymal elements. Diffuse lymphocyte infiltration with lymphoid follicles was detected in the stroma. What is the most likely diagnosis?

Explanation

 Basedow’s disease (grave’s disease) or toxic diffuse goiter is an autoimmune disease of the thyroid. Morphologically is characterised by a change in epithelium from prismatic to cylindrical and presence of foamy colloid.

 

Hashimoto’s thyroiditis is a destructive autoimmune thyroiditis leading to hypothyroidism. Characterised by lymphocyte infiltration of the stroma.

177. An 8-year-old girl presents with signs of disturbed twilight vision. This condition is caused by the deficiency of vitamin:

Explanation

 

Vitamin A/ retinol / retinoic acid is responsible for good vision. Vitamin E- tocopherol is an antioxidant. Vitamin K is for blood coagulation.

178. A 25-year-old young man came to the doctor complaining of general weakness, rapid fatigability, irritability, reduced working ability, and bleeding gums. What vitamin is likely to be deficient in this case?

Explanation

Ascorbic acid or vitamin C helps in the formation (crosslinking with the help of lysyl oxidase) of collagen fibres in fibrous tissues, teeth bone, capillaries etc. a lack of these vitamins can lead to small (petechial) bleeding especially in gums; increased breakdown of connective tissue etc
179. A 52-year-old man presents with fever and pain in the joints. Both of his fi- rst metatarsophalangeal articulations are deformed, swollen, and reddened. Blood urea is high. The patient is diagnosed with gout. What is the main developmental factor in the pathogenesis of this disease?

Explanation

 

Gout is a disease caused by increased production and accumulation of uric acid (hyperuricemia). Allopurinol is a drug indicated for its treatment. It competitively inhibits the receptors of Xanthine oxidase( an enzyme responsible for the conversion of hypoxanthine and xanthine into Uric acid). hyper azotemia refers to increased ammonia in the blood.

180. A person with the fourth blood group (genotype IAIB) has in erythrocytes both antigen A controlled by allele IA and antigen B controlled by allele IB. This phenomenon is an example of the following gene interaction:

Explanation

 

Semi dominance ( incomplete dominance) is seen when an allele ( a variant) partially dominates the other . Complete dominance is seen when an allele completely expresses its traits over another. Codominance occurs when the two alleles fully express themselves. Complementarity is seen when two genes contribute in a certain characteristic.

181. Vascular endothelium is characterized by high metabolic activity and synthesizes vasoactive substances. Among these substances there is a potent vasodilator synthesized from L-arginine. Name this vasodilator:

Explanation

Nitric oxide (NO) is a soluble gas continuously synthesized from the amino acid L-arginine in endothelial cells by the constitutive calcium-calmodulin-dependent enzyme nitric oxide synthase (NOS). It is a potent vasodilator. Histamine is also a vasodilator which is produced from mast cells.
182. A victim of a traffic accident was received by the intensive care unit. The patient is in a grave condition that can be characterized as a severe pathologic process that leads to exhaustion of vital functions and puts the patient into the marginal state between life and death due to critical reduction of capillary circulation in the affected organs. The patient is in the state of:

Explanation

Coma is a deep state of unconsciousness; shock presents with systemic hypoperfusion ( low circulation) in vital organs.It is defined as acute circulatory failure with inadequate or inappropriately distributed tissue perfusion resulting in generalised cellular hypoxia.
183. A person becomes less receptive to pain in physically or emotionally straining situations due to activation of:

Explanation

Nociceptive system is the part of the nervous system that responds to painful stimuli. The antinociceptive system is responsible for blocking the detection of painful and harmful stimuli.
184. A patient suffers from posttraumatic hemorrhage that resulted in development of hemorrhagic shock. What volume of circulating blood was lost by the patient?

Explanation

Hemorrhagic shock is a form of hypovolemic shock in which the body losses about 25-40% of blood.
185. A patient with pulmonary tuberculosis is prescribed the most effective antituberculous antibiotic. Name this drug:

Explanation

 Drugs used in the treatment of tuberculosis and their side effects include:

  1. Rifampicin- red urine and body fluids

  2. Isoniazid- neuritis ie, numbness of feet etc

  3. Pyrazinamide- gout, liver toxicity

  4. Ethambutol- optic neuritis and kidney related issues

  5. Streptomycin-  ototoxicity

 

Isoniazid and rifampicin are the most active anti-tuberculosis drugs.

186. During surgery performed in abdominal cavity a surgeon located the ligament of liver stretching from the anterior abdominal wall (navel) to the inferior surface of liver. What ligament is it?

Explanation

 

The round ligament of the liver stretches from the anterior abdominal wall (navel) to the inferior surface of the liver. The falciform ligament is found between the right and left lobe of the liver.

187. A patient has disturbed blood supply to the anterior papillary muscle of the left ventricle. What cardiac artery supplies this muscle with blood?

Explanation

 

The papillary muscles are a group of muscles that make up the heart mass, they are located in the ventricles of the heart.They include; The anterolateral muscle and the posteromedial medial. The anterolateral muscle is supplied by the left anterior descending artery (ramus interventricularis anterior) and the obtuse marginal branch of the left circumflex artery. The posteromedial muscle is supplied by the posterior interventricular artery. N/B the posterior interventricular muscle is likely to rupture due to blood supply from only one source.

188. Autopsy of the body of an elderly man revealed yellow spots and streaks in the aortic intima and white-yellow protruding plaques in the area of aortic bifurcation. Microscopy (with hematoxylin and eosin staining) shows round cavities in the thickened aortic intima. The cavities color orange when stained with sudan 3 and are surrounded with overgrown connective tissue. What process developed in the aortic intima?

Explanation

 

Yellow spots and streaks, white-yellow protruding plaques, cavities in aortic intima that turn orange when stained with sudan 3 all indicate fatty deposition in this organ which is as a result of metabolic disorder of cholesterol and cholesterol ethers.

189. An 18-year-old patient has developed candidiasis after the case of pneumonia treated with β-lactam antibiotic. What antimycotic agent should be prescribed?

Explanation

anti-mycotic drugs are medications used against fungal infections eg, Fluconazole, itraconazole etc. Beta lactams are drugs that inhibit the synthesis of bacterial cell wall. They include penicillins, cephalosporins, monobactams and carbapenems.
190. Preventive vaccination against poliomyelitis is made with inactivated vaccine introduced parenterally. What immunoglobulins create the postvaccinal immunity in this case?

Explanation

Immunoglobulin A (IgA), which is found in high concentrations in the mucous membranes, particularly those lining the respiratory passages and gastrointestinal tract, as well as in saliva and tears.

  • Immunoglobulin G (IgG), the most abundant type of antibody, is found in all body fluids and protects against bacterial and viral infections.

  • Immunoglobulin M (IgM), which is found mainly in the blood and lymph fluid, is the first antibody to be made by the body to fight a new infection.

  • Immunoglobulin E (IgE), which is associated mainly with allergic reactions (when the immune system overreacts to environmental antigens such as pollen or pet dander). It is found in the lungs, skin, and mucous membranes.

  • Immunoglobulin D (IgD), which exists in small amounts in the blood, is the least understood antibody.

 

IgG and IgM are give post vaccination immunity.

191. During pregnancy a woman has been taking an antiepileptic drug - sodium valproate. It resulted in her child developing a vertebral column malformation - spina bifida. Name the described effect of the drug:

Explanation

 

Teratogens are substances that disturb the normal development of a fetus or embryo eg, alcohol, drugs, infections etc. In this case, the antiepileptic drug- sodium valproate had a teratogenic effect causing spina bifida ( a malformation of the spine).

192. A patient was prescribed atropine sulfate to treat intestinal colic. What concomitant disease can be a contraindication for this drug?

Explanation

 

Glaucoma is an optical disease that leads to the damage of the optic nerve, it is characterised by increased intraocular pressure. It is caused by either poor flow/drainage of intraocular fluid (aqueous humour) or its increased production/accumulation. Atropine is contraindicated in this pathology because it leads to an increase in intraocular pressure.

193. A patient has developed status epilepticus. What medicine should be used in this case to stop the seizures?

Explanation

 

Diazepam is a tranquilizer, it possesses sedative effect and is used in epileptic seizures. Diprazin is a first generation antihistamine.

194. A 48-year-old man is unconscious. He has a history of several syncopal episodes with convulsions. ECG shows deformed QRS complexes unconnected with P waves, atrial contractions are approximately 70/min., ventricular contractions - 25-30/min. Name the type of arrhythmia in this case:

Explanation

Complete atrioventricular block is also known as third degree atrioventricular block. It is characterised by an absolute absence of conduction between the atria and ventricles. The first degree block is characterized by a prolonged PR interval. Second degree has two types; Mobitz 1- pathology lies in the AV node. Mobitz II- pathology lies in the distal conducting system ie, bundle of his and purkinje fibres.
195. In human organism significant blood loss leads to decreased blood pressure, tachycardia, and weakness. Eventually the sensation of thirst appears. What hormone participates in the development of this sensation?

Explanation

 As a result of reduced renal blood flow, the juxtaglomerular cells release Renin which converts angiotensinogen (produced from the liver) into angiotensin I. Angiotensin I is acted upon by angiotensin converting enzyme (present in the lungs) and is converted to angiotensin II. Angiotensin II is a potent vasoconstrictor and helps increase blood pressure. Angiotensin II stimulates the release of aldosterone from the zona glomerulosa of the adrenal cortex. Aldosterone is responsible for water and sodium reabsorption. 

 

N/B Angiotensin II has the following effects on the brain -Firstly, it binds to the hypothalamus, stimulating thirst and increased water intake. Secondly, it stimulates the release of antidiuretic hormone (ADH) by the posterior pituitary. ADH, or vasopressin, acts to increase water reabsorption in the kidney. Finally, angiotensin II decreases the sensitivity of the baroreceptor reflex.

196. After a trauma the man has lost skin sensitivity on the lateral surface of his forearm. What nerve of the brachial plexus provides sensitivity of the affected area?

Explanation

The musculocutaneous is responsible for skin sensitivity. Ulnar nerve- medial part of forearm. Radial nerve- lateral part of forearm.
197. Autopsy of a 50-year-old woman, who had been suffering from systemic connective tissue disease, reveals small dense kidneys with lumpy surface. Microscopy of the renal glomeruli detects there foci of fibrinoid necrosis, hyaline thrombi, hematoxylin bodies, and so-called ”wire loop thickening” of the basement membranes. What disease can be characterized by such changes in the kidneys?

Explanation

 

Systemic lupus erythematosus ( characterised by malar/ butterfly rash) is an autoimmune disease classified under type 3 hypersensitivity ( immune complex). Triggered by environmental issues, the body quickly produces antibodies against these antigens and its own tissues forming an antigen-antibody complex. These complexes are deposited in the kidneys which leads to an inflammatory reaction in the glomerulus ( glomerulonephritis) thereby, destroying the nephron. N/B a ‘wire loop thickening’ of basement membrane describes the microscopic changes that occur in this process.

198. A 4-year-old child presents with numerous carious cavities and yellowcolored teeth. The mother has a history of antibiotic treatment during her pregnancy. What antibiotic was the most likely taken by the child’s mother?

Explanation

One of the side effects of tetracyclines eg doxycycline is staining/discoloration of teeth.
199. A patient developed pyoinflammatory process of periodontal tissues caused by activation of the microorganisms inherent in the body, which are a part of oral mucosal microflora. What type of infection is it?

Explanation

 

An autoinfection is one that is caused by the normal bacteria flora present in our body. An Exogenous infection is caused i by microorganisms from our environment. In co-infection, the individual has two seperate infections eg TB and typhoid. A mixed infection is one which is caused by many microorganisms. A superinfection is an infection imposed while another infection is in a period of treatment.

200. After a hand fracture the man has lost the ability to touch his little finger with the thumb on the affected hand. Examination of the palmar surface shows one of the palmar muscles to be atrophied. Name the affected muscle:

Explanation

 

Opponens pollicis muscle is one of the three thenar muscles (abductor pollicis brevis and flexor pollicis brevis are the other). It is a small triangular muscle of the hand whose function is to oppose the thumb. N/B Abductor pollicis brevis abducts the thumb. Flexor pollicis brevis, which lies next to the abductor, flexes the thumb.